Get Mystery Box with random crypto!

NEXT| NEET PG| INI- CET Prep Club

Logo of telegram channel aiimsneetpgprepclub — NEXT| NEET PG| INI- CET Prep Club N
Logo of telegram channel aiimsneetpgprepclub — NEXT| NEET PG| INI- CET Prep Club
Channel address: @aiimsneetpgprepclub
Categories: Education
Language: English
Subscribers: 2.28K
Description from channel

Best place for PGMEE exams prep. with daily quiz, mnemonics, imp notes, and all the content req. for your dream Rank.

Ratings & Reviews

3.33

3 reviews

Reviews can be left only by registered users. All reviews are moderated by admins.

5 stars

1

4 stars

1

3 stars

0

2 stars

0

1 stars

1


The latest Messages 9

2021-07-02 19:10:11 10. A 61-year-old man has a non-ST-elevation MI and is admitted to the coronary care unit. The following day, he develops bradycardia but no symptoms. His blood pressure is 126/84 mm Hg, pulse 50/min, and on examination, the heart sounds are normal, with no extra sounds or rubs. His ECG has changed. Which of the following ECG findings is the best indication for this patient to receive a pacemaker?
39 views16:10
Open / Comment
2021-07-02 19:10:10 9) A 22-year-old primagravida woman develops hypertension at 28 weeks. She is asymptomatic and the examination is normal except for 1+ pedal edema. Her complete blood count, liver enzymes, and electrolytes are normal. The urinalysis is positive for proteinuria. Which of the following is true for this type of hypertension?
26 views16:10
Open / Comment
2021-07-02 19:10:10 7) A 67-year-old man presents with an anterior myocardial infarction (MI) and receives thrombolytic therapy. Three days later, he develops chest pain that is exacerbated by lying down, and his physical findings are normal except for a friction rub. His ECG shows evolving changes from the anterior infarction but new PR-segment depression and 1-mm ST-segment elevation in all the limb leads. Which of the following is the most likely diagnosis?
32 views16:10
Open / Comment
2021-07-02 06:09:06 6) Which of the following antiarrhythmic drugs mediates its effect by interfering with movement of calcium through the slow channel?
73 views03:09
Open / Comment
2021-07-02 06:09:05 5) A 42-year-old man with known valvular heart disease develops a fever for 1 week. He appears unwell; findings include a pansystolic murmur at the apex that radiates to the axilla and a soft S1 sound. He has petechiae on his conjunctival sac, linear hemorrhages under a few finger-nails, and painful, tender, and erythematous nodules on some of the distal fingertips. Which of the following is the most responsible mechanism for these physical findings?
45 views03:09
Open / Comment